Paper C 2021 - First solution

We're still working on double checking the solution to Paper C 2021, but we didn't want to delay posting a solution any longer.  

My first impression was that the first half was a bit more work than the second half. The claims related to some fairly complex mechanical structures, so in the first half you spent quite some time getting a grip on things. The second half had more tricky features though, although perhaps a bit less work.  

A funny detail is that some of the dramatis personae of Paper C 2015 returned in this paper. In 2015 Mr. Eilasie Kacez of Sabela Sports Industries asked the attorney Ms Molly Dorsett Pauley to file an opposition against Winterwute Corp. Now in 2021, Eilasie Kaceth (the same person?) switched jobs, but again asks Ms Molly Dorsett Pauley to file an opposition against Winterwute Corp.


We found the following attacks against the claims in the first part

Claim 1: Not novel against A3

Claim 2: Not inventive against A3 + A4 + A6

Claim 3:  Not novel against A3

 

For the claims in the second part

Claim 4: Not novel against A5

Claim 5: Not inventive against A4 + A5

Claim 6 +1 : added subject matter, no disclosure for spacers and 35% of RZCH

Claim 6+5 + range 17-23 has partial priority from P2. Is not inventive against A4+A5

Claim 6+5 + range 23-35 has the filing date. Is not inventive against A2+A6

A difficulty with claim 4 are the two features "a structure providing buckling resistance" and the "anti-buoyancy means".  However the double steel wall filled sandwiching the concrete seems to provide the anti-buckling function. I cited A3[02] as evidence in support of this.  The concrete also compensates for buoyancy so it is an anti-buoyancy means. I noted in the comments, that there many suggestions for an alternative inventive step attack on this claim. 

Claim 6 is an or-claim, one part of which can be attacked with added subject matter (6+1). The other part seems a first for Paper C, as it can claim partial priority. Given the worry of the client in his letter, I suppose you need to do two attacks on claim 6+5 to counter the possibility that part of claim 6 might remain. 

A2 has a tantalizing section with tourists sneaking a peak from a half finished device. I couldn't see how to utilize this though. 


We look forward to your comments!

Comments are welcome in any official EPO language, not just English. So, comments in German and French are also very welcome!

Please do not post your comments anonymously - it is allowed, but it makes responding more difficult and rather clumsy ("Dear Mr/Mrs/Ms Anonymous of 02-03-2021 22:23"), whereas using your real name or a nickname is more personal, more interesting and makes a more attractive conversation. You do not need to log in or make an account - it is OK to just put your (nick) name at the end of your post.

Thanks!

NB:  You may also want to check the comments to our first impressions blog, which was posted immediately after the exam and before this blog with our solution was posted , see here.

[Update 7 March 2021:] A copy of the paper, together with the claims for part 2, is available here.. 

Posted by Sander van Rijnswou

========================================================================

Some analysis blocks


Prepared by Jelle Hoekstra and Tanja la Cour Hoekstra


Effective dates of the claims (to be handed in)

Information provided by client:

Annex 1 claims priority of the applications NO20150000333 and NO20150000355. NO20150000333 has the following parts of Annex 1: [1] to [11] of the description and figures 1 to 3. NO20150000333 has only one claim and it is identical to claim 1 in Annex 1.

NO20150000355 is identical to the following parts of Annex 1: paragraphs [1] to [22], claims 1 to 5 and figures 1-5. Annex 1 was granted for European application EP16180339.8, which was filed containing all parts of NO20150000355 and additionally [23] of Annex 1.

Claim 6 was added during examination.

Claim 6 is of particular concern to us because we want to commercialize our devices using an elastomer containing not only 17 to 23 % by weight of RZCH but also, and in particular, containing 23 to 35 % by weight of RZCH.

Claim 1(indep): Basis in NO 333 (already as a claim), entitled to the priority of NO 333.

Claim 2(1), 3(ind), 4 (ind), 5(4): Were present as claims in NO 355 and not in NO 333. Have a basis in [12]-[22]. This was first described in NO 355. Entitled to the priority of NO 355.

Claim 6: added during examination. It requires that the elastomer contains 17-35% RZCH. Covers two embodiments (‘OR’ in the dependency)

First embodiment: claim 6(1) “elastomer in spacer in between reservoir and anti-buoyancy means”.
Basis for the composition of the elastomer in the spacer is in A1 [11], where it shows 13-47%. This gives no basis for the claimed narrower range. An objection is raised under Art.100(c), Art.123(2) EPC. No date is given to this claimed embodiment.

Second embodiment: claim 6(5) “elastomer in seal in between the segments”.
Basis for the composition of the elastomer in the seal is
A1 [22] >=17%, pref 23% and
A1 [23] alternative range 23-35%

In line with GL H-IV 2.4, this gives a basis for the range from the lower value 17 to the highest mentioned value 35. Thus no added subject-matter.

Of this composed range, the complexity lies in that the part 17-23% is disclosed in [22], present in NO 355 And that the part 23-35% in only present at filing of A1. This thus leads to a nice partial priority (first time ever in paper C).

Thus we have to split claim 6(5) into two (conceptual) parts:
a) 17-23% entitled to priority of NO 355.

b) 23-35% entitled to the filing date of A1.

Evidence list (to be handed in)

A3, A5 and A6 are published before the first priority date and can be used under Art.54(2) EPC for all claims.

A4 is a European application published after the first priority but before the second priority. A4 can thus be used under Art.54(2) EPC for claims entitled to the second priority or having the filing date, being claims 2, 3, 4, 5, and 6.

A4 has an effective filing date before the first priority and can be used under Art.54(3) for claim 1.

A2 is an excerpt from an article published on 06.08.2015 which lies after the second priority date but before the filing date of A1. The content of the article was discussed a week earlier, also in the same period. This can be used under Art.54(2) to attack claims entitled to the filing date, being claim 6(5) 23-35% option.

Fig.1 of A2 was visible from a tourist barge at a shipyard of the Calypso Foundation in the harbour of Warnemouth in the last week of May 2015. This lies before the second priority date. The tourists could see that workers strung wire ropes through duct channels in the pieces of steel to assemble them into an elongated structure. The tourist were not bound to secrecy. Available to the public is also the part of Fig.1 which could be visually observed. The footnote of A2 makes clear that the number of strands of the wire cannot be observed. This prior use, as evidenced by A2, can be used to attack claims 2-6.

The footnote of A2 describes common general knowledge known since 1970. This can be used under Art.54(2) for all claims.

Definitions, examples, properties (analysis, part of it to be used in your answer)

  • Reservoir A1 [02] is any compartment surrounded by a wall, out of which compartment water can be pumped out and into which water can be let in.
  • Electromechanical components necessary for underwater hydroelectric energy storage A1 [02]: an electric motor 14, a pump 15, a turbine 16 and a generator 17.
  • Buckling resistance A1 [04] means that at the deployment depth the reservoir does not collapse irrespective of the amount of water pumped out of the compartment
  • Anti-buoyancy means A1 [05] Anti-buoyancy means ensure that the device as a whole does not rise irrespective of the amount of water pumped out of the compartment. Known are anchors attached to the seabed, or ballast whose weight provides a downward force. A1 [09] The anti-buoyancy means of the first embodiment comprise holding means  and a ballast body 27 connected thereto. The weight of the ballast body 27 provides a downward force. Preferably, the ballast body 27 is made of concrete and rests on the ground 22 of a body of water
  • First reinforcing arrangement A1 [13] may be a framework of reinforcing beams
  • Second reinforcing arrangement A1 [15] may be a stiff mesh of steel bars
  • Tensioning tubes A1 [19]: duct channels or boreholes or other types of cylindrical cavities extending lengthwise within the walls of the reservoir sections.
  • Reservoir sections A1 [21] may be implemented as pipe segments
  • Epipelagic depths A2 P.3, i.e. depths down to 200 m below sea levelShallow depth A1 [14]
  • Mesopelagic depths A2 P.3, i.e. depths between 200 and 1 000 m below sea level. Lower depth A1 [14]
  • Devices which store energy underwater: subsea storage tanks for petroleum products, hydroelectric power plants. A2 p.3, l.10
  • Gasket is a sealing layer A5 [06]
  • Bumper: elastic spacer A5 [11]


Feature-effect pairs  (analysis, part of it to be used in your answer)

A1 [05] Anti-buoyancy means ensure that the device as a whole does not rise irrespective of the amount of water pumped out of the compartment.

A1 [10] Holding means facilitate deployment and enable servicing because the reservoir can be brought into and out of the holding means as needed.

A1 [11] Spacers are made from an elastomer to reduce the impact if the reservoir is
inadvertently moved against the spacers

A1 [13] First reinforcing arrangement:
the forces arising from the hydrostatic pressure are at least partly counterbalanced. The beneficial effect is a reduction of the net resulting mechanical stress on the reservoir's wall; A1 [14] the range of usage is extended to lower depths (for instance depths greater than 200 m below sea level), where the higher hydrostatic pressure allows more energy to be stored than at said shallow depth

A1 [15] Second reinforcing arrangement as a stiff mesh of steel bars 34 laid into the ballast body if the latter is made of concrete - improves stiffness, so the ballast body is able to better withstand any bending strain it is subjected to; A1 [16] allows safe deployment of the device in locations where the ground of the body of water is uneven.

A4 [10] reinforces ballast (concrete with stiff mesh of steel bars) for resting on uneven seabed [11] -> D2 for claim 2

A1 17] Using more than 7 strands requires a more complicated braiding technique, which is considerably more expensive.

A1 [19] Internal tensioning system of wire ropes strung through tensioning tubes is inherently protected from outside damage. A1 [21] The internal tensioning system of wire ropes guarantees a long service period.

A5 [07] A wire rope rigging as in the tank of figures 1a and 1b is preferred because the wire ropes are shielded against external damage so that a long service period can be expected -> D2 for claim 5

A1 [21] Reservoir sections 10 may be pre-manufactured at low cost.

A1 [22] Sealing layer should comprise an elastomer to provide a watertight joint. For long-term stability against deformation, the elastomer should contain at least 17 % by weight of RZCH. Preferably, the amount of RZCH is 23 % by weight or less. This means that the 20 elastomer is not very rigid and is well suited to shallow depths.

A4 [04] A gasket made from an elastomer may be used to ensure a watertight

connection between adjacent pipe segments. For sufficient long-term stability against
deformation, the elastomer should contain at least 10 % by weight of RZCH. The
inventors have found that the content of RZCH has to be 20 % by weight or less.
5 Otherwise the elastomer is very rigid and may not be watertight

A1 [23] 23-35%. Stability at greater depth

A6 [12] a higher amount of RZCH to be used in the elastomer for the sealing layer, for instance 30 % by weight of RZCH results in improved long-term stability against deformation so that the reservoir can remain in service for longer -> D2 for claim 6(5) 23-35% option


Overview of the technical field in the claims and prior art



========================================================================

Selection of the closest prior art

Prepared by Tanja la Cour Hoekstra

Claim 1

Claimed embodiment = underwater energy storage device.

Citable prior art = A3, A5 and A6 under Art. 54(2) and A4 under Art. 54(3).

No novelty attack possible in light of A4, as it misses the protrusion along the external surface of the reservoir and holding means releasably engaging with said protrusion.

A3, A5 and A6 all disclose an underwater energy storage device and are therefore all potentially suitable as CPA should none of them be novelty destroying.

A3 is novelty destroying for claim 1. No need for an additional inventive step attack.

 

Claim 2

Claim 2 depends on claim 1, but the field is limited compared to claim 1, as the underwater energy storage device now needs to be suitable for depths greater than 200 m below sea level.

Claimed embodiment = underwater energy storage device suitable for depths greater than 200 m below sea level.

Citable prior art = Fig. 1 of A2 and A3-A6 under Art. 54(2).

A2, fig. 1 does not provide a useful disclosure as it would not be possible to derive from visual inspection that it is an underwater energy storage device as evidenced by that the correspondent asks if the steel structure recently assembled on the shipyard would be a high-pressure submarine for investigating deep seas (A2, page 2, lines 1-2).

A3 relates to an underwater energy container array (A3, title and [1]). The device disclosed in A3 is not suitable for depths greater than 200 m but A3 encourages an adaptation in order to achieve the necessary buckling resistance (A3[11]). So, A3 can be made suitable.

A4 relates to a device for epipelagic depths (A4, title) and is unsuitable for mesopelagic depths (A4[8]). Accordingly, A4 is not suitable for depths greater than 200 m. Furthermore, the reservoir of A4 cannot be easily detached from the ballast pad by releasable holding means and requires severe structural modification. Accordingly, A4 is not suitable as CPA.

A5 is an underwater storage tank for energy in fossil form and teaches away from having protrusion along external surface (A5[12]). Accordingly, A5 is not suitable as CPA.

A6 is a low-cost one-time deployment reservoir (i.e. different purpose as it does not need service) and it is structurally very different. Accordingly, A6 is not suitable as CPA.

A3 must be CPA!

 

Claim 3

Claim 3 depends on claim 1, but the field is limited compared to claim 1, as the underwater energy storage device now comprises the electromechanical components necessary to render the underwater energy storage device an underwater hydroelectric energy storage device (A1[2]).

Claimed embodiment = underwater hydroelectric energy storage device.

Citable prior art = Fig. 1 of A2 and A3-A6 under Art. 54(2).

A2, fig. 1 does not provide a useful disclosure for the same reasons as mentioned under claim 2.

A5 is not a hydroelectric energy storage device and is therefore not suitable as CPA.

A3, A4 and A6 all disclose an underwater hydroelectric energy storage device and might therefore all be suitable as CPA, should none of them be novelty destroying.

The expected attack seems to be a novelty attack in light of A3, although A3 does not fully disclose a plurality of devices as defined in claim 1, as such devices each have a ballast body. Thus, a plurality of devices according to claim 1 would also provide a plurality of ballast bodies. However, the embodiment of A3 where several reservoirs are connected to a common pump, motor, turbine and generator, only has one ballast body (A3[6]). We nevertheless believe this is the intended attack.

 

Claim 4

Claimed embodiment = underwater energy storage device formed of sections.

Citable prior art = Fig. 1 of A2 and A3-A6 under Art. 54(2).

A2, fig. 1 does not provide a useful disclosure for the same reasons as mentioned under claim 2.

A3 does not have a device formed of sections is therefore not suitable as CPA.

A4-A6 all disclose an underwater energy storage device formed of sections and might therefore all be suitable as CPA, should none of them be novelty destroying.

A5 is novelty destroying for claim 4. This attack has caused quite some discussion among both candidates and tutors, since A5[10] states that “the tank’s walls are not built to resist forces arising from hydrostatic pressure”, which was seen by many as an indication that the device of A5 does not have a structure providing buckling resistance. However, in the next sentence, A5 states that “the tank will not collapse, irrespective of the amount of water pushed away by the fluid”, which fits the definition of buckling resistance given in A1[4]. In this interpretation, the structure providing buckling resistance is seen as the pipe letting seawater in and the impermeable membrane separating the compartment in the tank (A,[8] and [9]).

 

Claim 5

Claim 5 depends on claim 4, but the field is limited compared to claim 4, as the underwater energy storage device now comprises the electromechanical components necessary to render the underwater energy storage device an underwater hydroelectric energy storage device (A1[2]).

Claimed embodiment = underwater hydroelectric energy storage device formed of sections.

Citable prior art = Fig. 1 of A2 and A3-A6 under Art. 54(2).

A2, fig. 1 does not provide a useful disclosure for the same reasons as mentioned under claim 2.

A3 does not have a device formed of sections is therefore not suitable as CPA.

A5 is not a hydroelectric energy storage device and is therefore not suitable as CPA.

A4 and A6 both disclose an underwater hydroelectric energy storage device and might therefore both be suitable as CPA. Both A4 and A6 seem to have all features except the tensioning tubes through which wire ropes comprising 7 or less strands of twisted metallic wires are strung. A4 is closer as it already has lengthwise cavities in the concrete wall (A4[3]), where lengthwise cavities are examples of tensioning tubes (A1[19]). A6 on the other hand  teaches away from having any cavities in the concrete (A6[3]).

Accordingly, A4 is the closest prior art.

 

Claim 6, 17-23 wt%

Claim 6 is dependent on claim 5 and claimed embodiment is the same as for claim 5.

For this part of claim 6, the same prior art is citable as for claim 5 and A4 continues to be CPA for the same reasons as for claim 5.

 

Claim 6, 23-35 wt%

Claim 6 is dependent on claim 5 and claimed embodiment is the same as for claim 5.

For this part of claim 6, A2 is citable in addition to the prior art citable against claim 5. We probably have to use that for something! And indeed, if comparing A4 and A2, A2 is closer than A4 as it has all features of claim 6 except the amount of the filler RZCH in the elastomer of the sealing layer.

Accordingly, we have a classical C-exam swap of CPA from the parent claim to the dependent claim as CPA for this claim is A2.



Comments

  1. Good evening Sander.

    was A5 disclosing "strands of metallic wires and the number of strands being 7 or fewer" ?

    I didn't find these features in A5.
    So why is A5 then novelty destroying for claim 4 ?

    A2 discloses in the footnote wire ropes PI-R have 7 strands of twisted metallic wires or fewer (known since 1970). Exactly like in the claim 4!
    On top: A5 is already using this kind of PI-R wire ropes, see par 005.

    I mean, A5 + A2 seems to fit perfectly together, am I missing anything ?

    Thanks !

    ReplyDelete
    Replies
    1. I would argue that the footnote of A2 is just evidence of common General Knowledge in the technical field since it is known from the 70s. Hence we are not actually taking any teaching from A2 itself.

      Delete
    2. It was just the definition what some one skilled in the art would understand under the mentioned cable of A5.

      Delete
    3. I mean common knowledge of course ... The spectators could otherwise see only figure 1 , they could see no wire ropes and tensioning tubes and so on.. A2 also made clear what is epileptic and megapileptic depths :) But the footnote was crucial in my view..

      Delete
    4. A5 [05] had that they use PI-R rope. That rope has 7 or fewer wires. So A5 already discloses this, no need to combine to get the feature

      Delete
    5. Damn, I didn't spot that.
      But in these kind of situations they usually award points also for inventive step A5+A2, aren't they?

      However, what worries me more is the CPA switch in claim 5: from A5 to A4. Obviously, in order to incorporate the wire ropes of A5 into A4 one must cut through the "double wall made of steel sheets between which concrete is poured as a filling". Although "lengthwise cylindrical cavities" (="tensioning tubes") are incorporated inside the concrete of A4 one still must basically destroy the entire reservoir of A4 in order to add the wire ropes INSIDE the wall.
      Is that what a skilled man would do? (without hindsight, of course)
      I could understand replacing the screws by rivets or welding but the insertion of wire ropes inside the cavities of A4 is a NO GO, neither technically nor fictionally. And if this is the "right" solution then I see no point in doing this exam, it's just a time waste.
      Didn't sleep the entire night only judging my A5+A2+A3 combination for claim 5 (basically A5+A3).
      In fact I could have combined A5 with any of A3,A4 and the prior art disclosure of A1, they all disclose the arrangement with pump,generator,turbine etc

      Delete
    6. Hi again and sorry for maybe annoying ..
      I didn't understand your solution: "Claim 6+5 + range 23-35 has the filing date. Is not inventive against A2+A6".
      This claim is though dependent on claim 5 which is dependent on claim 4.
      For claim 5 the solution involves A4+A5. Shouldn't A4 + A5 be involved also in the solution of Claim 6+5 23-35 due to dependency?
      Or did you mean A4+A5+A2+A6 => Claim 6+5 23-35 ?

      Delete
  2. Eilasie Kacez must be a really vengeful person. And I wonder what kind of company Winterwute Corp is? They make snowboards and underwater energy storage devices

    ReplyDelete
    Replies
    1. ?
      https://williamgibson.fandom.com/wiki/Wintermute

      Delete
  3. I don't agree with attack against claim 4, because of the feature anti buckling feature. It was written in A5 that the wall cannot take any forces, but it's no Probleme since pressure inside and outside the reservoir are always the same.
    After I tried to combine A5 with A4, I realized, that it should rather be a combination of A4 with A5 since A4 is already anti buckling. The other features were easy to add, since A4 already mentioned to change the attachment means and A5 comprised to use bolts or cables. Additionally, A5 comprised the effect to secure the attachment means from environment what was exactly the effect mentioned in A1. It just made perfect sense at the end.

    ReplyDelete
    Replies
    1. But the cables must run within the tensioning tubes extending lengthwise within the walls of the reservoir. See A1 par 019. Embodiment 1 in par 003 of A4 discloses lengthwise cylindrical cavities, thus the tensioning tubes but one would not put wire ropes inside without hindsight. A4 + something seems not realistic to me.

      Delete
    2. I'm glad someone else has come to this conclusion, everyone else seems to have a totally different idea but I thought the same as you! I believe D5 even said the device didn't need anti-bouyancy means, plus it said the wire rigging was an alternative to bolts in D4 and D4 said you might want to change the bolts for an alternative tensioning means. I thought it was the most perfect attack xD
      It's interesting that everyone seems to have a very different idea of what was the right thing to do here, will be interesting to see how this paper is marked

      Delete
    3. I think regarding anti-buoyancy, A5 mentioned something regarding that it's made of concrete and that it is sufficient to prevent the vessel from moving up regardless of fluid being pumped in or out (or words to that effect). According to A1, that's the definition of an anti-buoyancy means. So in A5 it's passive rather than active, but it's still there.

      Delete
    4. Maybe some people understood the marking way a bit automatically, I mean: you see that claim 5 includes a pump etc .. releasing energy and then the prior art MUST be switched. In most cases works and I understand the point. But in this particular case it was more hassle to change A4 into claim 5 instead of adding the modules to the already existing stupid tank.

      Delete
    5. Mark: anti buoyancy was disclosed in 003 of A5. Anti buckling was defined in A1 004 and then the same definition pops up in 010 of A5. Nevertheless , A1 says that a reservoir must resist hydrostatic pressure. A5 says it does not resist hydrostatic pressure see 010 first sentences. But the claim didn't require the feature about hydrostatic but merely about buckling and in A5 there is definitely a little anti buckling ...

      Delete
    6. Hi John, yes I agree. At least I think I'm agreeing with you. I did a novelty attack on Claim 4 using A5. The anti buoyancy comment was in response to 21:19 indicating anti-buoyancy means weren't needed. Of course it's very difficult to get points across in comments! Any way, best of luck to all involved!

      Delete
    7. This was the only claim where I did not do the same as Sander. Also because of the anti-buckling feature. From an exam design point of view, it makes more sense to have Novelty A5 for independent claim 4, followed by inventive step A4 + A5 for dependent claim 5. But that's not a good reason for selecting attacks of course.

      The definition in A1[004] is quite clear "... according to this invention means ... does not collapse irrespective of the amount of water ... wall MAY BE designed ...". In A5[010], it is clearly explained that the wall will collapse when the tank is not filled and that the "walls are not built to resist forces arising from hydrostatic pressure". Also, A5 does not use the "marine-grade steel" of A3[002], which could make it buckling resistant.

      However, if I am wrong, I missed all the marks for a novelty attack. If Sander is wrong, he just did an unnecessary attack. He needed A4 + A5 for claim 5 anyway. So, as a candidate, I would feel better now if my answer did include the novelty A5 attack.

      Delete
    8. But Joeri, A5[010] says this "The tank will not
      collapse, irrespective of the amount of water pushed away by the fluid."
      And this is the exact definition of anti buckling in A1 004: "Buckling resistance according to this invention means that at the deployment depth the reservoir does not collapse irrespective of the amount of water pumped out of the compartment." So I would agree with Sander.

      Delete
    9. Reading it again, I see that I probably interpreted the claim in a too narrow way. It does not have to be the reservoir structure (i.e. the wall) that needs to be buckling-resistant. It is the structure of the underwater energy storage device. Therefore, the buckling resistance does not need to come from the wall material but can come from the membrane arrangement too. So Novelty A5 indeed. I lost some marks there.

      Delete
    10. Thanks for the quick reply. you had me worried for a second. All good. thanks

      Delete
    11. I agree that from a paper C point of view, it does make sense that claim 4 could be not novel over A5 and than claim 5 not inventive over A4+A5, so that we have to change the closest prior art. But, from a technical point of view, the water inside the device of A5 is not a structure which forms part of the device. A5 does not collapse as long as, in operation, it is filled with water/petroleum. But claim 4 does not say: "operatively configured for providing buckling resistance in use"; it says "comprises a structure for providing a buckling resistance". A structure is something part of the device, not the water inside it. I read that some people saw the membrane or the pipe of A5 as a "structure for providing buckling resistance", because they allow water to come into the device. But I am perplexed that a flexible membrane or a pipe may be interpreted as such structure, since they do not provide (directly) buckling resistance. Anyway I am curious to see what the Committee will say in this regard.

      Delete
    12. Johan Meulpolder6 March 2021 at 16:28

      The definition of "buckling resistance" is (A1[0004]): " Buckling resistance according to this invention means that at the deployment depth the reservoir does not collapse irrespective of the amount of water pumped out of the compartment"

      So the membrane allows this to happen and it forms part of the structure of the device. The structure of the device then is such that A5[010] "The tank will not collapse, irrespective of the amount of water pushed away by the fluid." almost verbatim A1[004]
      Either the term "buckling resistance" means something or it does not. We know what "buckling resistance" means through A1[004]. Use/not-in-use do not enter the equation.

      There would have been a problem, as Joeri rightly points out above, if it was not the device as a whole but the reservoir which was supposed to have a structure like this. But that is not the case. So A5 is novelty destroying for claim 4

      Delete
    13. A5 cannot be novelty destroying for claim 4.
      claim 4 discloses an underwater energy storage device, whereas A5 discloses an underwater storage tank which cannot produce energy.
      A5 discloses all the features of claim 4 (and 5) except the electromechanical portions thereof (pumps etc.)
      A5 is the CPA for claim 4 and 5 for an inventive step attack. A skilled person would be prompted to use the concept of A5 to create an underwater energy storage device, as confirmed in A2, where skilled persons were "inspired by old subsea storage tanks... although they cannot be used as hydroelectric power plants".
      that was my solution at the end, hoping it is fine.

      Delete
    14. Boudewijn Schoon6 March 2021 at 18:25

      Not quite correct Kavi. As you point out: A2 pg. 3 line 11-12 clearly states that "– in a way they are also devices which store energy underwater"
      So A5 discloses an underwater energy storage device as per A2.
      I think your confusion is that you seem to think that claim 4 is producing energy through electromechanical means. It is not. That is claim 5. Claim 4 is simply about storage.
      That's why A5 is novelty destroying for claim 4 but not for claim 5.

      Delete
    15. yes, now I got it. it was sneaky, and they got me.

      Delete
  4. A2 use disclosure seen by the tourists is a horrible trap if not novelty destroying for claim 4. You can see all the elements put together in Figure 1, which represents what they saw on the day. Nothing in claim 4 would be hidden from view if you can see the threading etc.

    ReplyDelete
    Replies
    1. it's indeed a trap I'm afraid. In A2 it's somewhere written that by seeing one cannot know which type of wire ropes is used (i.e.PI-R or PI-F relating to the number of strands of twisted metallic wires). Thus, A2 public on May is at least not novelty destroying to claim 4...(but maybe a CPA for claim 4??)

      Delete
    2. I am still not sure about this. A2 is not novelty destroying since you cannot see what types the wire ropes are. But wouldn't any skilled person just check what types of wires exist for such a set-up? It really doesn't seem very difficult to come to the subject-matter of claim 4 from there...

      Delete
    3. I argued lack of inventive step of claim 4 taking the prior use as CPA combined with A5 and the CGK in view of the footnote of A2... but I guess it's overkill if claim 4 not novel in view of A5 in any case

      Delete
    4. Why can't you see 7 strands? Seems visible to me. Don't need to know the wire brand, but can see 7 or less strands.

      Delete
    5. I did the same chris and it worked really well with the argumentation, I felt. So I'd actually be really a bit surprised if that was wrong. I'm still confused about a5. I thought about using it but I felt very unsure.

      Regarding the strands: I think you cannot see how many there are. Its not how many wire ropes there but of how many strands those ropes are composed of. The footnote in a2 states that it cannot be seen from the outside. That is at least my understanding.

      Delete
    6. Fear you may be right regarding the strands of the wires, but what a cruel trick for such a time pressured exam. Sometimes I do wonder what the point of these exams are.

      Delete
    7. The last sentence of the footnote on page 4 of A2 makes clear that you can't see the difference. But it's not a trap. It's a test to see if you understand that A2 is just prior use evidence for claim 4 and full prior art later for claim 6. The C committee does not set traps.

      Delete
    8. I thought also that since the interviewer asked if the thing that could be seen from the bridge was a submarine, it wS a hint the public couldnt tell it was an energy storage device.

      Mich

      Delete
    9. You can say "the C committee does not set traps" with all the post exam confidence/humour/cockiness you like, but to present an atypical disclosure as almost usable but not quite based on visibility is a nasty trick for an exam in my book, any day of the week. A2 was already an interesting document by way of it's usability against various claim, the additional fact it had a non usable disclosure was trick, and I'll struggle to be persuaded otherwise.

      I'd forgive any mistake related to Art54(3),partial priority etc. But I do think grey areas like "is this a real disclosure or not" is pushing fairness in such a time pressured exam. Oh a sentence says they couldn't quite make the exact aire thread out? Oh come one.

      The committee knew what they were doing.

      Delete
    10. Well Joeri, after digesting everything , I am not so sure about 🪤 traps. A5 seems indeed to have “anti buckling” only because it is full with fluid at any time. But in parallel A5 literally discloses also anti buckling according to the definition of A1. A5 says yes and no. Honestly claim 5 is inventive both with regard to A4+A5 and Vice versa.

      Delete
    11. I did an inventive step attack against claim 4. A2 prior-use + general knowledge (Editors footnote in A2). I considered that the device of A5 is not an energy storing device and no anti-buckling in there.

      Delete
    12. When you talk about 'traps', you assume that the exam committee is trying to set you up for a fail and that it is your task to discover the traps and not fall into them. I truly believe that this is not how the exam committees draft these exams. With the exam, they try to test candidates by offering them a plurality of legal issues and all information that is needed to solve those issues. It's the candidate's task to first identify the issue, then consider all relevant information, and finally apply the relevant law to the situation in order to come to an answer.

      For all important issues the exam committee really tries to ensure that, after you gathered all relevant information and if you understand the law, you do have very clear hints for what the answer should be. Unavoidably, candidates can miss or missinterpret some of the relevant information. Not because the committee tries to obscure the correct answer, but just because that's what happens when you have to process a lot of information and solve a lot of issues in a limited amount of time. With all the practice I've had over the last 15 years, it still happens to me too. Luckily, you only need 50 marks. It is ok (and even expected) to miss a few important points.

      Delete
    13. Dear Joeri
      Please don't pretend the time management has to do with real life situation.
      We all work. It's even a requirement to take the exam.
      A work like this would be invoiced 7 working days.
      Best regards

      Delete
    14. @Anonymous 6 March 2021 at 16:26

      I don't see where I did pretend this.

      Delete
    15. Nobody means traps in the real sense (on purpose) but traps resulting from the fictional way of structuring the exam.

      Delete
  5. Thanks Delta! My question is: in claim 3 we have a plurality of devices according to claim 1. Therefore we also have a plurality of anti-buoyancy means. How can the single pedestel of the alternative with the single/common pump in A3 be considered as a plurality of anti-buoyancy means? Hence, do you consider equally valid say that claim 3 lacks inventive step (not novelty) in view of A3?

    ReplyDelete
    Replies
    1. I saw the same issue and I agree with you in that A3 does not disclose a plurality of ballast bodies and thereby not a plurality of anti-buoyancy means. We cannot be sure what the expected attack is and if marks will be awarded for both attacks.

      Delete
    2. I don't think that claim requires a plurality of anti-buoyancy means. It requires a plurality of energy storage devices that all have an anti-buoyancy means. It's not a problem if they all share the same anti-buoyancy means.

      Delete
    3. Thanks Joeri. I disagree. Claim 3 requires a plurality of all device elements mentioned in claim 1. If we consider that claim 3 requires a plurality of devices that all have the same anti-buoyancy means, then we consider that the anti-buoyancy means is not a real part of the device of claim 1, but only something that the device is connected to. For example, would a plurality of devices that have different anti-buoyancy means but a common container, also satisfy claim 3? Would a plurality of devices that have different containers but a common holding means and a common antibuoyancy means also satisfy claim 3? All these devices would be modified versions of the device of claim 1. How can we have a plurality of devices according to claim 1, but not have a plurality of each of the elements mentioned in claim 1? It is for this reason that I propose that claim 3 should be best attacked as lacking invetive step in view of A3.

      Delete
    4. And to be a bit more mathematical: the device of claim 1 comprises (x+y+z), where z is the anti-buoyancy means. A plurality of devices, let's say N devices, according to claim 1, should comprise (Nx+Ny+Nz). A3 referring to the case of N devices connected to a common pump, only discloses that a first device has (x+y+z) (i.e. a first device according to claim 1, where z is a large pedestal) and the rest N-1 devices comprise only x+y and are connected to to z of the first device. Hence, A3 does not describe (Nx+Ny+Nz), but only describes (x+y+z)+(N-1)(x+y). Clearly this is not a plurality of devices according to claim 1. In novelty attacks all the features of the attacked claim must be found in the prior art document. If there is a feature missing, e.g. if the number of certain elements is different, then we enter the realm of inventive step. The sharing of a common antibuoyancy means, which is identical to saying that only one device has antibuoyancy means and the others do not have but are connected to the anti-buoyancy means of said one device, is a clear deviation from the literal interpretation of the phrase "plurality of devices according to claim 1".

      Delete
    5. It's an interesting discussion, but I just followed the exam flow there.

      Each storage device does have its own holding means (part of the anti-buoyancy means). It's only the ballast body that is shared. When I look at Fig. 2 or 3 and combine the pedestals into one larger pedestal, I still see a plurality of energy storage devices. It's the reservoirs that store the energy.

      The paper does not provide any nice arguments that you could use for inventive step. A1 does not provide any additional information on claim 3. A3 gives a disadvantage for a large pedestal, but that information was already used for the inventive step attack on claim 2.

      When the novelty argument does not appear unreasonable and there is no clear argumentation provided for an inventive step attack, they probably expect the novelty attack. This does, however, not mean that an equally reasonable inventive step attack will not give you marks.

      Delete
  6. Hi Sander,
    many thanks for the quick solution.
    Could you please explain your reasoning regarding Cl6+5 and partial priority?
    In my opinion, [23] discloses that 23 to 35% is "alternative to", ie sth different from, 17 to 23% disclosed in [22]. Both ranges have different effects - low stiffness vs high stiffness. I tend to the view that there is not sufficient basis for 17 to 35%. Maybe it was intended otherwise by the Exam Comittee, but "alternative" really points away from a 17-35% range.

    ReplyDelete
    Replies
    1. I did partial priority similar way. Partial priority is to treat the claim as a generic OR claim of the two parts of the range. It is then treated as a normal OR claim: first part of the range and second part of the range are both attacked independently. So whether both parts have the same or a different effect does not matter. My 5 cents.

      Delete
    2. The term “alternative” was for me a very strong pointer to treat it as an OR claim of two alternatives.

      Delete
    3. Thanks for the quick solution! I am also really interested to hear the explanation for why there is a partial priority situation here. For example, it is not straightforward in my opinion that [22] discloses the range 17-23.

      Delete
    4. The prerequisite for any kind of priority claim be it partial or not is that the subject matter in question has basis in the application as filed, which is very questionable here imo

      Delete
    5. From a different angle:
      If the range of [22] is regarded as embodiment A and the range of [23] is regarded as embodiment B. Paragraph [23] discloses expressis "Alternatively ... embodiment B". Would a claim drawn to A+B have basis? I don't think so. Why would then the range find basis in [22] and [23]?

      Delete
    6. Hi, I struggle with same question - is there even base for claim 6 in application as filed? [22] and [23] disclose alternatives. So in my eyes, claim 6 is undisclosed combination of values from separate embodiments

      Delete
  7. Partial priority in D1 and in C? Seems not well-balanced, too much weight on a single legal concept in this year’s exam. And the Guidlines are way too poor on it, just the cryptically formulated headnote of the Enlarged Board decision, no guidelines how to apply it.

    Michael

    ReplyDelete
  8. Alexander Bogander5 March 2021 at 21:34

    Hi

    The DP answer says there is an issue with claim 4: 'a structure providing buckling resistance', however this is resolved by what is said in A1 para 4: 'buckling resistance according to the invention means that at the deployment depth the reservoir does not collapse irrespective of the amount of water pumped out'.
    This feature is clearly disclosed in A5, para 10.

    Do you not agree?

    ReplyDelete
    Replies
    1. That's what I said above. I fully agree. But I don't understand where are the features about the 7 strands or fewer.. are they in A5 ??

      Delete
    2. in A2 actually, there is a footnote about common general knowledge of PI-R wires, so A5+A2 or A5+CGK in my oppinion

      Delete
    3. Yes, I guess so. It follows from A2’s cgk from the 1970s that the type of wires in A5 has 7 or fewer wires

      Delete
    4. I meant it was a difficulty, not an issue. I also though this was disclosed in A5.

      Delete
    5. i suppose cgk is not limited to one document so you cant say A2's cgk... its just cgk. so maybe novelty using A5 is fine.

      do we know if we get marks for having alternative or slightly incorrect attacks? or do we get zero..

      Delete
    6. I also argued that A5 has a structure providing buckling resistance, because it it says in A5 it doesnt collapse, as defined in A1.

      However, the problem I had was that A5 said did not collapse irrespective of amount of fluid pumped out, in A1 it defined water not fluid...not a problem in real world, but in C usually cannot equate water=fluid ;-)
      ignored it though and put cl. 4 lacks novelty over A5

      Delete
    7. Alexander Bogander6 March 2021 at 21:29

      @Sander my apologies for misquoting you!

      Delete
  9. curious to know if we will get marks for saying inventive step attack against claim 3, using the first and second embodiments of A3? rather than a simple novelty attack. I thought this would be a more robust analysis, even though the second embodiment is mentioned to comprise of all of the first embodiment. also, on a similar point, would it have been necessary or sensible to seperate the first and second embodiments of A3? there was no explicit disclosure of a plurality of devices of the second, and so perhaps the argument would be that because the forst embodiment discloses a plurality of devices, the skilled person would use that teaching to further extend it to the second embodiment

    ReplyDelete
  10. disagree about claim 4 against novelty of claim 4. A2 has not been used for any attacks! Golden rule is that if you haven't used a document then there is something wrong going on. I agree with all the other attacks though

    ReplyDelete
    Replies
    1. A2 is used against the part of cl. 6 that hasnt any valid priority

      Since A2 was published after both priority dates, I think it is kind of clear that it can only be used against this subject-matter (and from a tactical point of view, it is clear that it must be used, because otherwise, A2 would only be good for definitions and as you point out that is usually not the model solution)

      Delete
  11. well, I’ll see everyone next year for C. I’ve got to be honest, I made a dumb mistake with wires and A2. If I had just slowed down and not panicked, I could have done much better. Advice to future candidates (and me in 2022)!

    ReplyDelete
  12. This comment has been removed by the author.

    ReplyDelete
  13. This comment has been removed by the author.

    ReplyDelete
  14. If I re-write the definition of buckling resistance “Buckling resistance according to this invention means that at the deployment depth the reservoir does not collapse

    due to the hydrostatic pressure

    irrespective of the amount of water pumped out of the compartment”
    Would you guys agree?

    One thing that bothered me about only citing “reservoir not collapsing irrespective of the amount of water pumped out” is that one would wonder why would the reservoir all of a sudden break? Is there a quality problem or something?
    I think this question is critical to clarify if A5 is novelty destroying for claim 4

    ReplyDelete
    Replies
    1. Given the old exam, I think A5 should be used against novelty of claim 4, because then you can use A4 as CPA for claim 5 and you have a shift of CPA...that was a frequent concept in old C paper that CPA shifted for some dependent claims

      Also A5 clearly stated that it would not collapse, if fluid was pumped out, which is per defintion a structure providing buckling resistance

      Delete
    2. Well, I am still feeling you are quoting out of context about the reservoir not breaking during water pump.

      Suppose A5 is novelty destroying, that would mean any pump that is not breaking when water is pumped out is buckling resistant, which is normally the case if the reservoir in use is not too crapy.

      It's not about how the old exams are designed. I am still confused if one could simply generalize by taking one piece out of a whole picture: buckling resistance without hydrostatic pressure.
      Enlighten me please.


      Delete
    3. My point was a tactical one. My approach to the paper is finding out, what they intended, not arguing as in real life.

      They want to test, whether you grasb to concept of prior art. Typically they provide some dependent claim that has some CPA that is different from the novelty attack on the claim it depends on. So for some claim in the exam, it makes sense that you have a novelty attack on an independet claim and then an IS attack on a dependent claim based on different prior art.

      This fits the perfectly the situation of claims 4 and 5. Claim 4 is not limited to electrical energy, whereas claim 5 is. THis makes A4 CPA for claim 5. If you attack novelty of claim 4 based on A5, you have your shift of CPA. It is not an argument that A5 is novelty destroying for claim 4, it is just a check whether the situation A5 against claim 4 and A4+A5 against claim 5 would fit. It fits. Therefore, it seems that it was what was intended.

      Of course, you still need to argue that the features of claim 4 are disclosed in A5. However, A5 clearly states that the reservoir will not collapse and A1 provides the respective definition for buckling resistance. I think that is it.

      Forget what it would mean in the real world. These papers work by a certain logic. If you apply this logic to the present one, it makes sense. This years C paper was longer than before and involved more text, but in terms of structure, I think it worked pretty much as the papers in the past ten years or so.

      How would you take one piece out of the whole picture and generalize? A5 clearly talks about hydrostatic pressure, which is avoided by letting water into the device and having the membrane which separates the two chambers. This design (membrane and letting water in) keeps the internal pressure at the same level as the external pressure, which avoids collapse of the reservoir by simply avoiding hydrostatic pressure, which is a means for providing buckling resistance. The claim is not limited to buckling resistance of the material of the container, "means for providing..." can be anything.

      Delete
    4. All you say it is true. But the same logic of C committee wants you not to destroy a device on the way improving it. If you take A4 as CPA for claim 5 because it already discloses the stroring-release_module then you must deal with the following DIFFERENCE: how to incorporate the wire ropes disclosed in the structure of A5 into the walls of A4? This is only possible by destroying the reservoir of A4. All the items disclosed in the annexes (reservoirs etc) exist and must be combined somehow in order to arrive at SM of different claims. Manufacturing a new A4 reservoir incorporating the wire ropes like in A5 is pure HINDSIGHT.

      Delete
    5. Hi John, unfortunately A4 is the CPA for claim 5. You wouldn't have to destroy A4 because A4 [003] discloses: "The concrete filling can be solid or can comprise lengthwise cylindrical cavities in order to reduce the weight"

      These lengthwise cylindrical cavities are tensioning tubes as per A1. Check A1[019] for the definition of tensioning tubes. "Tensioning tubes may be implemented as duct channels...or other types of cylindrical cavities extending lengthwise within the walls of the reservoir sections."


      Furthermore, it is not hindsight as A4 is already begging for an alternate solution: A4 [005] "However, the exposure to the outside environment means that there is risk of damage to the screws, for instance when deploying the modular reservoir. To avoid this, other tensioning methods may be used."

      Delete
    6. Seems right, unfortunately for me..

      Delete
    7. A4 had lengthwise cylindrical cavities, A4 had screws but taught that there is risk of damage and other tensioning methods can be used.
      A5 taught the wire ropes, it taught them as an alternative to screws, it taught that they are shielded against external damage

      So basically A4 has feature that is described as detrimental and A4 give inducment for replacing the feature, A5 discloses the feature of the claim, teaches it as equivalent to the respective feature in A4 and provides the technical advantage

      From the logic of paper C, the attack A4+A5 fits perfectly well, all the information can be put together smoothly. That is why I think that was the model solution. Mostly in paper C, you need to determine which documents can be combined to have all features of a claim and then for the selection of CPA, you can check if the information is in the annexes that makes the attack simple, usually that is the attack they are looking for...however, they certainly only give marks for different attacks. I think what you need is a sound argument as to why something is CPA and as long as you dont just write "AX is CPA because it has most features in common with the claim" or something like that, you can score high marks, even if they were looking for a different attack.

      Delete
    8. Thanks anonymous, your advice eases my concerns a little bit, maybe I am lucky and get some points .

      Delete
    9. But still... if you have a look at figure 1 of A4 you won’t see any holes. This means the holes are inside and not all the way along the pipe segments 21. Which means further the skilled one would have to drill the holes himself in order to make them suitable for the wire ropes of A5. Pls Compare with figure 2b of A5 and you’ll understand what I mean.

      Delete
    10. A4 says "The concrete filling can be solid or can comprise lengthwise cylindrical cavities in order to reduce the weight"

      A1 says ". Tensioning tubes may be implemented as duct channels or boreholes or other types of cylindrical cavities extending lengthwise within the walls of the reservoir sections."

      => "lengthwise cylindrical cavities" = "Tensioning Tubes"

      A4 has tensioning tubes as required by claim 5

      As simple as that. Do not overthink it, do not apply technical reasoning. Paper C gives you all the information and usually you have to simply follow the definitions of the paper.

      Delete
    11. your judgement looks perfect from the committee point of view. BUT "lengthwise cylindrical cavities" are only emphasised in the text of A4 and when you look at the figure you may understand that yes, they are there but they are not having open ends such that the wire ropes protrude from one end to the other end of the entire reservoir through and connecting the pipe segments. This seems a bad trap, for sure the committee didn't realise this little detail .

      Delete
    12. If there were holes shown on figure 1 of A4 I would not hesitate a second choosing A4 as CPA.

      Delete
    13. I think you are applying too much technical reasoning ;-) ... paper C is simply very artificial

      Nevertheless, open ends may be drilled or the cylindrical cavities can be made with open ends. I would not think about the necessary changes to a pre-existing reservoir, but rather the adaptions to a technical teaching, i.e., a theoretical concept of a reservoir. If by adapting the manufacturing process you can easily introduce cylindrical cavities with open ends, I think that even by applying technical reasoning, there is nothing against modifying A4. I think it is less relevant, whether you can easily adapt a pre-existing actual reservoir.

      Delete
  15. What I dont understand about the A5 novelty attack on claim 4 (aside from the anti-buckling feature which I dont believe is disclosed) is that claim 4 is an underwater energy storage device. The tank of A5 is not suitable as an eneegy storage device unless you consider gasoline a form of energy?

    ReplyDelete
    Replies
    1. It was explicitly stated that tanks such as in A5 may be considered energy storage devices in A2...this type of definition typically matters in C papers and should be taken for granted....

      also in real life, a tank for gasoline is some form of energy storage, is it not? The point here is that claim 4 is not limited to electrical energy (other than claim 5, which also leads to shift of CPA to A4 for claim 5)

      Delete
  16. thanks delta. I have one question, if i attack claim 4 using A4+A5 for IS, can i also get some points, or not at all?

    ReplyDelete
    Replies
    1. That's the right thing to do. If I am not mistaken, in 2016 C paper, for one claim, there is one novelty attack plus one inventive step attack for one independent claim.
      That inventive step attack was use for subsequent dependent claims.
      In fact, the inventive step attack had more points than the novelty attack

      Delete
    2. thank you very much for your quick reply.

      Delete
    3. The examiners report always state that points are also awarded for attacks that are not part of the model solution. As long as it is well reasoned, I think you can get points for any attack, although presumably only the model solution will attract full marks.

      In some papers, the model solution comprised both novelty attacks and attacks for IS, but I think in those cases the novelty attacks were based on 54(3) prior art, which is basically a weak attack since it can easily be circumvented by incorporating some trivial feature into the claim

      I also had A5 as novelty-prejducicial for claim 4 and since it is 54(2) prior art, in case the attack is correct, I would assume that the model solution only has this attacks, but you may certainly get points for A4+A5, since it can certainly be argued (in fact for claim 5 I have A4+A5, so it would apply to claim 4 as well)

      Delete
  17. Was anyone else confused about the A2 document where it said "last week" in Italics - although published in August or so, the Italics to me implied that it was also in relation to what happened in May and the information was being relayed/oral disclosure in May during the interview....I just found the mention of Last week to be confusing. I know am totally wrong but did find that confusing though and whether it was a trick to mess with our heads again like all the other papers.

    ReplyDelete
  18. Definition of Buckling pressure from https://www.sciencedirect.com
    When a tube is subject to an external pressure, it can collapse if the pressure exceeds a critical value. This is termed as a critical buckling pressure. In this case, the external pressure also means the difference of the internal and external pressures if the latter is greater than the former.

    ReplyDelete
  19. For claim 6 how would the amount of elastomer be enabled in A2. They only saw the Figure? The interview was not prior art.

    ReplyDelete
    Replies
    1. Sorry I meant to say the wires. How are the wires known? Just A2+A6+common general knoedge? A2 also doesn't have an electrical module attached based on the figure. It also implied that the interviewer thought it was a submarine and not an energy device, so surely not a sensible CPA? Id like some further elaboration on this please (just for interest!!!)

      Delete
  20. I used the "tantalizing section with tourists" (i.e. the disclosure through public display as witnessed by A2, which I labelled V2, for German "Vorbenutzung") to form an alternative attack on claims 5 and 6(5;17-23). Main main attacks on these were also A4+A5, but alternatively, A4+V2 also worked.

    ReplyDelete
  21. I attacked claim 4 lack of inventive step using A5 and cgk from A2 - feasible? Cgk used for Inventive step. I can see why Novelty using A5 only now but hoping I get something for this?

    ReplyDelete
  22. A3 is not novelty destroying for claim 1 or 3.

    Para. 7 says the second embodiment _may_ have all the features of the first embodiment

    Para. 8 says bumpers _may_ be mounted..


    Para. 10 says the bumpers _may_ be made of an elastomer

    This is multiple selections

    ReplyDelete
    Replies
    1. All features were shown in the embodiment of Fig. 3 right?

      Delete
  23. A2 can not be used for what the public saw. They saw some lumps of metal and ropes. That is all. It could have been a ship being made. The ropes could have been anything.

    It is only the later article that discloses relevant technical features

    ReplyDelete
  24. Is it the _structure_ of A5 that provides anti buckling?

    It appears to be water inside it that provides anti-buckling

    ReplyDelete
    Replies
    1. I argued it was evidenced by A4, par. 3 (A5 has the same walls as A4), that A5 was actually providing anti-buckling

      Delete
  25. Or the petrol provides anti buckling if it's full of petrol with no water. Us this a structure?

    ReplyDelete
  26. I used A4 + A5 for C5. I thought electric energy storage purpose meant it was CPA
    For C6 - I found A5 +A6 to be a better combination.

    I also did an Art 54(3) EPC on C1 and

    A4+A6 for C2 as A4 purpose was under 200m sea level.

    Any thoughts

    ReplyDelete
    Replies
    1. I should say A4 +A5 (with A2 cgk)

      Delete
    2. As you correctly noted, device of A5 is not suitable for hydroelectric energy conversion purposes. Hence A4 is CPA for claim 5. Since claim 6 depends on claim 5, they are still referring to a device for hydroelectric energy conversion. Hence,in my opinion, A5 cannot be a CPA for claim 6.

      Delete
    3. I also concluded that A5 could not be used as CPA for claim 5, based on A2, page 3, comment by Costaud. However, I did A6 + A5 (distinguishing feature being the tensioning tubes and wire ropes)

      Delete
  27. I put an inventive step attack against C4 starting from A2 public disclosure. The skilled person would have seen all of the features of C4 but the type of cables used (there were lots of definitions in A1 that could be used to explain that the skilled person would have understood the use of the double wall as an anti-buckling feature, and the skirt as a ballast if filled with rubble - I don't remember the exact words). Conveniently, the CGK in A2 also explained that you could not tell which cables they used by just looking at them. The SP however would know that the PI-R cables would have been suitable, either from the CGK or by combining with A5, which used PI-R cables. One thing is certain, the statement in A5 that the device did not have any resistance to hydrostatic pressure was baffling.

    ReplyDelete
  28. @A2 I did the same with A2! I saw the footnote which "only" specifies the type of wire rope cannot be known by external inspection... so I thought, other things were to public on May then?? Also, other features such as reservoir sections/anti-buoyancy means/underwater energy storage seem to me all "visible" from appearance anyway... so I use A2 (CPA) + A5 (wire PI-R) for claim 4 and then for claim 5, simply A2 again + (wire PI-R) + (sealing layer) with a reasoning on separate partial problems...

    And again, the first interview question fails to hint that one cannot know it's a submarine or power plant from inspection... If it has been written somewhere clearly that from outside one definitely cannot know if a submarine or a power plant has been built at shipyard, then I would not consider A2 for claim 4..

    I really don't understand why they could not put everything clearer..under great time pressure+ huge panic, we need everything as clear as possible for C!!!

    ReplyDelete
  29. Jean-Nicolas Héraud6 March 2021 at 08:48

    I've presently presented exactly the same solution as Sander (however partially due to lack of time).

    ReplyDelete
  30. Thanks for your help and solutions! In the stress of the exam the partial priority issue didn’t occur to me, so I analysed claim 6 as if the whole thing had the filing date as effective date, and I analysed it as partial problems for each half of the range (as I noticed that the problem being solved was slightly different for each half), using A2+A5 and A2+A6. Given that this all occurred due to me missing the priority issue (and therefore error carried forward) do you think I can still get some marks for this analysis? Thanks

    ReplyDelete
  31. What was the relevance of the limited number of designations in the patent under attack and in the prior art applications?

    Mike

    ReplyDelete
    Replies
    1. I would say there is none.

      For Art. 54(2) EPC it never mattered.
      For Art. 54(3) I think an overlap in designations was required under EPC1973, but under EPC2000 it doesnt matter anymore.

      The priority of A4 was not valid (or at least it was outside the 1 year), that also didnt matter. I think they just put in random for the prior art.

      Delete
  32. Why is A3 CPA to claim 2? I mean, A3 was not suitable for big depths and required modifications. So, the purpose of A3 is not the same as the purpose of claim 2. I used A6 as CPA...

    ReplyDelete
    Replies
    1. Yes I dismissed A3 as CPA for claim 2 because it wasn’t suitable for deep water. I’m confused about that..

      Delete
    2. I did the same. A3 need adaptation so it was not suitable for lesopleagic depths. A4 was the closest prior art for claim 2.

      Delete
  33. The solutions proposed by the tutors and the annals are carried out without the constraint of time.
    However, the real difficulty of the paper is the lack of time, in often a distant technical subject.
    This was accentuated this year by the provision of 7 documents to study in a 3 hours C1 test where it was 5.5 hours before. And the success rate was already very low. Total paralysing panic was the immediate consequence.
    It is doubtful whether the EPO's promise to offer the same paper as before was fulfilled.
    If I am asked to write my name, and the copy is taken after a second, is it fair to say that I can't even write my name?
    This paper is unfair. It would be useful to know why.

    ReplyDelete
    Replies
    1. In my homeland, the passing rate is around 10% per year. I happened to have a chance to have lunch with one of the examination committee who also owns a law firm with 200-300 employees. He told me that they do make the exam very hard on purpose; otherwise there will be too many patent attorneys on the market. He was also joking that in this way he could also save money for his own firm by not paying much for his patent engineers... I guess EPO would like to start to play that game too? have waited for 2 years to take EQE2021 which led to nothing for me...I'm thinking maybe I should go to IT industry where one gets paid a lot easily, at least much more easily than passing EQE2021!

      Delete
    2. If they want to take this route, the non-native speakers are clearly disadvantaged since they can have more difficulty in mastering the language skills/reading speed as the native speakers

      Delete
    3. I don't think they care about that. There is an epi online course where they show answers of C paper 2015-2017 from several candidates who failed and also give comments about those answers. They have commented with something like: if this candidate is a native speaker he would have passed or giving more time will help non-native to pass, etc. This shows that they EXACTLY know non-natives are disadvantaged. In view of paper B & C this year, it's clear to me that they don't give a shit about the disadvantages of non-natives at all. Well, I guess they don't even give a shit about what candidates complain or think until candidates become real patent attorney. Cruel world ;-)

      Delete
    4. Agree with Anonymous 9:49 “ It is doubtful whether the EPO's promise to offer the same paper as before was fulfilled.”

      Promised in ‘ Information on the schedule for the EQE 2021 examination papers ‘. Quote:

      “ Paper B - 4 March 2021
      Paper B will have the same syllabus and character as before. Paper B lasts 3.5 hours.
      Candidates will be allowed to print the prior-art documents and the drawing(s), but none of the following: the description and claims of the application, the EPO communication, the client's letter and the amended claims. The documents allowed for printing will be made available approximately ten minutes before the start of the examination.”


      Delete
    5. Wrong quote, copied B section rather than C, apologies. (But is holds even more so for paper B!!!)

      Correct quote:

      “ Paper C - 5 March 2021
      Paper C will have the same syllabus and character as before, but it will be split into two parts.“

      Delete
    6. Legal action can be taken positively.

      Delete
    7. I'm non-native. Here is my advice to other non-natives: don't waste time to complain and think that anybody would care you're disadvantaged. It's your responsibility to improve your EN, DE or FR level to something like native. Looked at D1.1 this year, DE natives have advantages of more than 30mins than others. EPO says, they guarantee that no candidate will be disadvantaged as a result during the marking process.

      Well, how?? how could they measure the advantages given to the DE natives with extra 30 minutes and without any influence on their focus, and how could they measure the sufferings of non-DE speakers by worries, panic, stress during the D1.1 part which greatly affect their focus and performance on D1.1 or even on D1.2 and D2 during the rest of day?? NOT possible. It's an unfair game, really unfair, but you need to find your own way to play with that.

      Delete
  34. The paper itself was ok. It was the splitting that caused the mess.

    Paper C is usually constructed out of 2 different objects. The objects have cross-linked features that resemble each other, but are for different functions. This is of course done on purpose.

    For attacking one embodiment you do not merely need the positive features for the attack. It is even more important to be able to quickly ELIMINATE what is NOT relevant. This is done by quickly mapping features to either one or the other object. However, without knowledge of the second embodiment, the quick mapping cannot be performed. As a consequence, one has to go through all prior art one by one and test its relevance for the embodiment under attack. This is very time-consuming, if not distracting and confusing.
    Below I will give one example, but I have more.

    This was different in Mock 2-paper C, wherein only the claims for the 2nd part were missing. The description and figures were present for both embodiments.
    In Mock 1 the subject-matter was much simpler, and therefore some common sense could be used for elimination.

    When there is written "the spacers may be of elastomer" and later "the seal may be of elastomer", in a trained candidate alarm bells start to go off: Same material, different function, different location! When you then read the prior art and come across the word 'elastomer' or synonyms thereof, or the function, one immediately maps those features to either of the two options. So a prior art mentioning a gasket of elastomer, which seals watertight, is discarded as irrelevant for claim 1 within 3 seconds.
    But without information of both embodiments, any occurrence of elastomer in the prior art deserves serious scrutiny. For example, part 1 of paper C only mentioned that the spacers are of elastomer. No alarm bells. So when the prior art mentions that a seal is made of elastomer, it appears worth considering if a seal can be used as spacer. Indeed, there was also prior art disclosing a seal-like ring around a cylinder which was a bumper, i.e. a spacer, so it did seem possible.
    Obviously this analysis leads to a dead-end, and on hind-sight it is utterly nonsense. But when the usual alarm bells are de-activated (due to removed subject-matter in the split format), it costs 10 minutes to draw that conclusion, instead of 3 seconds.

    In conclusion, in part 1 we missed mapping- and anchoring points. We missed reasons to eliminate, and we missed alarm bells. Leaving us with a lot of unanchored noise to plough through.
    It is then impossible to work efficiently towards a clear goal. In the end candidates come to the right answers, but with a lot of extra time and effort.

    So my tip for next year to the drafters:
    -if you split paper C: design it for that purpose, and make sure there are sufficient coordinates, anchor points, warning signs etc.
    - don't split paper C.
    - other option is to keep subject-matter very simple. Just compare:
    A corkscrew with a straight part, a helical part, and a handle.
    An underwater hydroelectric energy storing device with wire ropes strung through a tensioning element. (or so)

    Please consider this for next year :-)






    ReplyDelete
    Replies
    1. I could not agree more. I understand that for everybody including the C committee it has been a tough time. But, I have the clear impression that the committee only has cut the claims into two parts and supplied the client's letter in two parts. No adaptation of the claims or prior art seems to have happened. The paper was, in my view, excessively long and for some candidates not on the technically easy side. A short overview. C 2017: 26 pages; 2018: 33 pages; 2019: 30 pages. Here 2021: 39 pages.
      It is expected in all exams that candidates can reasonably efficient discard irrelevant information. Lets say that half of the text is irrelevant. Here for the first part, you got all information and much more than normal where then only 25% is relevant. Then it would have been nice/reasonable if you could have easily discarded one or two annexes as irrelevant for the first part. But, no. As you indicate for the elastomer the information is very confusing if you have not seen claim 6.
      Clearly, with minimal effort the committee could have done something. E.g. put claim 6(1) as claim 4 in the first part. To allow for some time for the additional, the novelty attacks on A3 could have been simplified. Better would have been to simply remove the elastomer or its composition for the spacer.
      Clearly, this exam was a bad start for an online exam. In my opinion, more effort can be expected of the committee.

      Delete
    2. Thanks Lonneke and Jelle for your comments. I totally agree with you. Having a blind part in C-1 brings in a whole lot of confusion. Having the whole picture of the subject matter to attack appears essential for a smooth conduct of C.

      Examination committee has been introducing 3 new concepts in C this year:
      - split modality (and online exam)
      - non-enabled prior use
      - partial priority in ranges

      A C paper where two different categories of claims are tested would allow a much smoother identification of discartable prior art.

      Delete
    3. How could they feel justified in their tactics to mess with candidates careers, livelihood and mental states. It is shocking and beyond belief. No other examining committee would do such a thing.

      Delete
    4. For the first part, I think A2 and A5 were only relevant for some definitions. That should have been plain obvious to the comittee.

      It would have been very easy to move/copy these definitions to other parts of the paper and supply A2 and A5 only for the second part. That would shortened the amount of prior art for the first part significantly.

      Since it is so obvious, I really do not understand, why they did not do it.

      Delete
    5. Exactly! With hindsight, when you knwow all the claims and all the prior art, it is easier to decide which information to discard.

      Delete
    6. I may not be in the best position to judge the quality of the split because I only did a quick and dirty analysis after the exam. However, my impression is that the split was arranged quite well. Before the break there is more material to become familiar with but the attacks were more straightforward. The more difficult issues (partial priority, complex prior use in A2, does A5 read onto claim 4) came after the break when candidates were already more familiar with the subject-matter and the documents.

      The halfway break will never be timed optimally for all candidates. Not everyone processes the information at the same pace and in the same order. Candidates who prefer to do a more thorough analysis of all information before they start answering are disadvantaged by the introduction of a break, regardless of where exactly the break is positioned. In my opinion, the large amount of information was a bigger problem than the position of the split. All prior art had 3 pages of relevant text. This larger amount of information adds to the same problem for the same candidates.

      A break halfway is difficult to avoid if you don't want candidates to be glued to a screen continuously for 5.5/6 hours. The amount of information to process is easier to adapt, and should have been adapted.

      But we don't know yet how this will affect the results. You don't need to attack all claims to pass the exam. Some candidates will miss points because they could not properly attack all claims. Other candidates may have attacked all claims, but miss points because they skipped important issues or too quickly made the wrong decisions. Only when we have the results, we can conclude whether the difficulty level of the exam was high/low/fair/unfair compared to other years.

      Delete
    7. Joeri,
      You are indeed right that you are not in the best position to judge :-)
      And I strongly disagree that the split was arranged quite well. It simply wasn't. In the first part, it was a nightmare to know which document (not to mention which embodiment of that document) is relevant to the limited claims. The second part, as Sander rightly points out, was easier to figure out in that respect, but had trickier features etc. I managed to do all right, but its a far cry from having an unsplit paper C in which you can make sense of things more naturally and easily. Its quite clear that paper C has become appreciably more difficult now. If they committee insists on retaining this split, I think it would be well advised to follow Lonneke's and Jelle's suggestions, especially as per Lonneke: "If you split paper C: design it for that purpose, and make sure there are sufficient coordinates, anchor points, warning signs etc."

      Delete
    8. I completely understand that no split would have been easier for everyone except for the rare candidate who needed exactly 3 hours for part 1. But given the fact that a split was deemed necessary (for screen time reasons), do candidates have a feeling that the time pressure in part 1 was significantly different from part 2? Or was it just too much work and information in general?

      Delete
    9. To answer your question: part 1 was significantly different from part 2 as Lonneke has also pointed out. The information that we had to process for part 1 was simply too much. Part 2 was easier insofar as information processing is concerned, but was trickier and it had its own challenges as Sander has pointed out.

      Delete
    10. Joeri, it is not the amount itself that matters: 39 pages remains 39 pages, whether split or not. And it is not the points or the split in time.
      It is the processing of information that becomes flawed. Consider the following two situations:


      CLAIM 1: Blue car with black tyres.
      D1: a car which seems turquoise and with white tyres
      D2: a picture of tyres
      D3: a red car without tyres, and a picture of loose black tyres
      D4: an advertisment: paint your car any colour!

      Analysis for claim 1:
      D1 and D3 are cars. D1 is turquoise. Is turquoise the same as blue? Most probably, I will look for a definition later.
      But D1 has white tyres, so I need a document with black tyres. Perhaps D2 or D3? D3 has black tyres and is also a car. So D1 with D3 is promising.
      I only have to look up a definition that turquoise is the same as blue. Mmm, not in D1. Perhaps in the patent? Not either. Surely in D4 then? Neither. Oh oh, not good. Well, I can always use D4 for the blue colour. But then I am combining three documents? D1 and D3 and D4. I could still check D2. It also had tyres and perhaps it had the blue colour in. Then I could do D1 and D2. Let's just check to make sure. No blue in D2. This is trouble.
      D1 and D3 and D4 then for claim 1? Oh, hang on, there is still D3. Yes, that is quite good actually. D3 and D4. Pfew, finally I found the attack.



      Now consider having ALL information in one go:

      CLAIM 1: Blue car with black tyres.
      CLAIM 2: Turquoise car with exchangeable tyres.

      Analysis for claim 1:
      D1 is not blue and has white tyres, so two differences. Not promising.
      D2 is just tyres, probably to do with the exchange-thing in claim 2.
      D3: ah, promising, it has black tyres, so the only difference is that it is red.
      D4: paint, great!
      D3 and D4 it shall be. Let's start writing!


      You see how you need claim 2 for eliminating and filtering?

      Delete
    11. Impressed by your analysis Lonneke! I recognize exactly your points and I got into severe doubts and time trouble during the exam because elimination could not be done the would it should. If that was intentional, the C committee is moving to US-style attacks of randomly combining features without correct PSA, which they should absolutely not!

      Delete
    12. I think that one of the main problems is that the committee seems to expect candidates to only consider one claim at a time and to quickly put aside all information that is not useful for attacking that claim. I like that approach, but there is no reason why other approaches to the paper should not be possible and acceptable too. The exam tests if you can apply the law, not if you approach oppositions in some undisclosed apparently preferred way.

      The break position appears to be based on the amount of marks awarded for the different parts. However, many candidates spend more time on the first 50 marks than on the second 50 and therefore struggled with the first part. If there is no break, they will catch up later. This year that was not possible.

      For me, and probably for some candidates too, the second part was more work than the first part. As soon as I found out that only A3 was novelty destroying for claim 1, claims 2 and 3 followed very quickly. The two partial problems starting from A3 worked so well that I didn't see any reason to switch CPA. Part 2 took me much more time and I had to look at more documents much longer to come to my decisions.

      Just 15 minutes extra for each part is clearly not enough to allow different candidates to use different approaches and perform as well as they would in a paper without breaks. The obvious solution to this is to not have a break, but I don't expect that solution will be used.

      The other option is to make sure that it's just a shorter paper with less information, such that time pressure will not be an issue regardless of how you prefer to approach the paper. They have tried this before by adding half an hour extra, but this led to higher pass rates. Higher pass rates are good news, but they appear to reduce the incentive for the committee to limit the paper size. This year's paper was really longer than usual. I can't remember any other paper with 5 prior art documents having at least 3 pages of text.

      Delete
  35. I agree. Although I studied engineering, it still took me much more time to understand the subject-matter than previous years.

    ReplyDelete
    Replies
    1. A solution can also be to give the documents one month before the test, without the claims.
      Or give the anteriorities, without the patent. After all, the candidate is not supposed to be judged on technique. Everyone knows, however, that there is no equality between the candidates according to the proposed technical theme.

      Delete
  36. claim 5:

    A4 does not disclose the feature of "pump driven by an electric motor for storing energy and turbine driven by generator for releasing energy".


    claim 6(b) does not fulfil 123(2). You cannot simply merge two PRODUCTS having respective alternative ranges into one PRODUCT.

    You may argue that A4 refers to "necessary for underwater.." which is defined in A1. However, that definition of A1 does not disclose the feature as such; You need to ADD "for energy saving" and "for releasing energy" "driven by/drving" which are not part of that paragraph!!

    ReplyDelete
    Replies
    1. There is implicit disclosure of "pump driven by an electric motor for storing energy and turbine driven by generator for releasing energy", since it discloses the "necessary components" and A3 defines these components. I think it was intended as an implicit disclosure and they wanted to test, whether candidates know about implicit disclosure and recognize it.

      For claim 6(b) I am uncertain, however I think there is no Art 123(2) issue. It is not a merger of two products, the two ranges in [22] and [23] actually referred to one and the same product and were presented as alternative ranges, thus "17 to 23 or 23 to 35" was clearly disclosed and it is effectively the same subject-matter as "17 to 35"...Art. 123(2) is about subject-matter, not about wording

      Delete
    2. It can only be implicit, if it is the ONLY existing option to combine pump+electric motor + turbine; where it is written in the paper that it is the only option? in addition A3 cannot be used as reference for "common knowledge".

      Delete
    3. A1, par. 2 described this as CGK (Figs. 1 and 2 of A1 are prior art)

      Delete
    4. A3 and A4 had same proprietor. A4 referred to a common hydroelectric module (can't remember exact term). A3 defined this module with the exact language of the claim. A3 was usable as evidence of fact for definition of a common hydroelectric module.

      Delete
  37. to claim 6b:

    if the application discloses a first car which has V between 20km/h and 100km/h and an alternative car that has V between 100km/h and 200km/h, can you claim that you have one CAR that has V from 20 to 200km/h??? of course not.

    The case law is clear "The content of an application must not be considered to be a reservoir from which features pertaining to separate embodiments of the application could be combined in order to artificially create a particular embodiment (T 296/96, T 686/99, T 1206/01; T 3/06, T 1206/07, T 1041/07, T 1239/08, T 1648/11, T 1799/12, T 1853/13, T 1775/14)."

    ReplyDelete
    Replies
    1. You are right,

      yet [22] and [23] did not disclose a first car and an alternative car. It disclosed a car which has V between 20 and 100. THen it said, alternatively the car has V between 100 and 200. It was an alternative range for the same car. Thus it clearly disclosed the car with V 20-100 OR with V 100-200, which is effectively the same as car with V 20-200

      Delete
    2. No, sorry. You need to build a new car (or new sealing layers) in such a manner that can run in one of V between 20 and 200 (i.e. it is not effectively the same as the two other cars).

      From a pure numerical point of view they are as you said; however, we are dealing here with products that we want to sel and license... It is not a mere combination of intervals. As soon as you change the interval it is not any more the same car!! even if you refer to the same term "car" in the two paragraphs; because they are alternative cars.

      A1 discloses clearly TWO embodiments. The one described in [23] is explicitly named alternative embodiment meaning that [22] refers to EMB1 and [23] refers to EMB2. The case law does not allow a simple combination of EMB1 and EMB2 in one claim...

      Delete
  38. Hi Sander, thank you for your insights. For claim 3, I thought that A3 lacked enablement, because the enlarged pedestral in A3 allegedly required further modifications to make it suitable to rest on the seabed. For this reason, I combined A3 with A4, which teaches the 2nd reinforcing structure to provide enablement. What are your thoughts on this?

    ReplyDelete
    Replies
    1. I agree I did the same. A3 was not suitable for depth higher than 200 m. Could not be closest prior art.

      Delete
    2. I'm happy I'm not the only one. I did the same

      Delete
    3. Claim 3 was dependent upon claim 1. Did not need to be suitable for <200m.

      Delete
    4. @ Anonymous 16:05: I agree with you on that, but the uneven seabed was a common problem and not limited to either one of the depth. So A3 was not enabling for either one of the depths in my opinion

      Delete
  39. A3 could not be closest prior art for claim2 the last paragraph mentions that it needs to be adapted for going at a certain depth, meaning it is not adapted as it is. D4 was the closest prior art. And then you add A3.

    ReplyDelete
    Replies
    1. I think that last paragraph was an extra incentive for the skilled person to look for the adaptation to A3 to make it suitable for mesopelagic depths

      Delete
    2. Well that meant that what was disclosed in A3 was per se not adapted for mesopelagic depths, otherwise it would not need adaptation. So it cannot be closest prior art.

      Delete
  40. Claim 4 was totally novel against A5. A5 did not have a structure having anti bulking. See the definition in A1 and the last paragraph of A5. However it was not inventive over A4+A5

    ReplyDelete
    Replies
    1. this is what i said.

      Delete
    2. Me too
      + according to the definition of the invention [2], a reservoir is any compartment surrounded by a wall, out of which compartment water can be pumped out and into which water can be let in.

      In A5 the tank (= the reservoir as I understand it) is filled with a fluid that is lighter than water [3]=> therefore no water !! Further in paragraph [9] A5 talks about partioning the compartment in a lower and upper portion : in the upper portion petroleum is pumped in and out (as in the reservoir according to the invention) while the level of water adjust itself in the lower portion => therefore no pumpinf in/out

      Thus, I cannot understand a novelty attack by basing the argument that a reservoir according to claim 4 (defined by pumping water) is equal to the tank or the upper portion filled with petroleum with is pumped in and out, netiher that such reservoir according to claim 4 (defined by pumping water) is equal to the lower portion where water adjust itself, not pumped in and out.

      Delete
    3. I argued that A4, par. 3 is evidence that the device of A5 actually already had an anti-buckling wall, even though A5 said that the tank's walls are not built to resist forces arising from hydrostatic pressure (A5, par. 10).

      Delete
    4. The anti bulking properties are defined in A1 and specify that water can be pumped out without causing the walks to collapse. In A5, it was specifically disclosed the opposite, the walls would not resist. The only reason why it could be use at mesopelagic depths was that the liquid inside the tank was counter balancing the pressure caused by the water. Thus, D5 explicitly had no anti bulking.

      Delete
    5. The pipe 18 of A5 is a structure that provides buckling resistance to the tank. It passively let water in and out so that walls wouldn't collapse irrespective of depth. I seem to recall the para of A1 that defined antibuckling used the same 'irrespective' language that A5 used. It seemed a clear link.

      Delete
    6. Anti bulking was defined as preventing the walls to collapse even if the water is pumped out. In A5 the water had to come in in order to prevent the walks from collapsing. Thus if the water of A5 was pumped out, the walks would collapse. I think it was even stated. Thus the tank if A5 had no anti bulking structure.

      Delete
    7. Sorry I said anti bulking all the time but I meant anti buckling. I did the paper in Fr...

      Delete
    8. I know where you're coming from and I hung on this for ages. But A5 states "the tank will not collapse irrespective of the amount of water pushed away by the fluid". The fluid is pumped in or out by another pipe 17 with a valve.

      Delete
    9. if I'm not wrong, claim 4 says "anti buckling MEANS". in A5, the anti-buckling means is the membrane, that counter balance the pressure inside the reservoir together with the pipe allowing water to get in and out, that's why it will not collapse independent of the depth.

      Delete
    10. No it was anti buckling structure. After having a second thought I agree that it could be interpreted as a buckling structure.

      Now, there is still the issue of the reservoirs. A1 defines it as a compartment surrounded by walls from which water can be pumped in and out. In A5 the compartment where the water goes is surrounded by a walk. However, the water cannot be pumped in or out. It adjusts automatically. The compartment containing the fluid is surrounded by a membrane. So neither compartments if A5 corresponds to a reservoir according to A1.

      Delete
    11. Furthermore it is not possible to pump water out if the compartment surrounded by the membrane because it does not contain water.

      Delete
  41. With the online several exams sessions per year could more easily be organized.
    Why not?

    ReplyDelete
    Replies
    1. That would be a great idea...or even have different papers at different times of the year. The dissapointment of Paper B cannot be easily overcome overnight so that the next day one can be fresh for Paper C

      Delete
    2. Not even to mention the devastating effect of the German language disaster in D1-1. That ruined my D1 and made me very very uncomfortable and uncertain for the rest of the week. Estimate that that made me loose 5-10 marks in each of A, B and C.

      Delete
  42. am I the only one finding claim 3 novel in view of A3? The document does not disclose a "common" pump and turbine. I combined A3 with A6 last paragraph to attack the claim with A56EPC

    ReplyDelete
    Replies
    1. Yes I did the same but unfortunately a3 does disclose a common pump and turbine. I overlooked it :/

      Delete
    2. thx...strm
      would really appreciate it if you send me the paragraph in A3

      Delete
    3. Para. 6, second sentence

      Delete
    4. Same (A3 + A6).. Unfortunately missed the statement in par. 6 due to the last-minute rushing.

      Delete
  43. Few differences in my solution as compared to DP's:

    Claim 3: I thought A3 did not disclose a "common" pump and "common" turbine. Now see that I've missed it in par. 6 though ..
    So I did I.S., but no effect was mentioned. I therefore set "providing an alternative" as the OTP. Then, e.g. A6, discloses common electromechanical units (definition, A1, par. 2).

    Claim 5: I did I.S. A6 + A5 (distinguishing feature being the tensioning tubes and wire ropes).
    Reason being that A2 said that petroleum tanks (A5) were not suitable as electric energy storage devices.

    Claim 6(5)(17-23): A6 + A4

    Claim 6(5)(23-35): A6 + A5

    Any thoughts? Thanks.

    ReplyDelete
    Replies
    1. A2 also says that they were inspired by old subsea storage tanks... enough for me to make A5+common general knowledge.

      Delete
  44. One different question: Has delta patents any estimate how many points may be given for the legal questions? I assume there must be some in the pool due to two priority dates + Art 54 (3) + partial priority + discussion of document A2...

    Any estimate possible?

    ReplyDelete
  45. Anyone else do A5 + A6 for claim 2.

    ReplyDelete
  46. Can I get a sanity check: the model solution says that claim 6(1) is added subject matter, since there's no disclosure for spacers and 35% of RZCH. However, paragraph [0011] of A1 states that "the spacers are made from elastomer... the elastomer contains 13 to 47% by weight of RZCH"

    ReplyDelete
    Replies
    1. sanity check: the spacers in par 11 are the bumpers, not the sealing layers. they are two different things.

      Delete
    2. To have no added subject-matter issue/priority issues, it needs to be the same in scope. Not broader, not narrower. 13-47 gives no basis for 35.

      Delete
    3. Yes - the range of 13-47 does not disclose the intermediate values, such as 35

      Delete
  47. A5 cannot be novelty destroying for claim 4.
    claim 4 discloses an underwater energy storage device, whereas A5 discloses an underwater storage tank which cannot produce energy.
    A5 discloses all the features of claim 4 (and 5) except the electromechanical portions thereof (pumps etc.)
    A5 is the CPA for claim 4 and 5 for an inventive step attack. A skilled person would be prompted to use the concept of A5 to create an underwater energy storage device, as confirmed in A2, where skilled persons were "inspired by old subsea storage tanks... although they cannot be used as hydroelectric power plants".
    that was my solution at the end, hoping it is fine.

    ReplyDelete
    Replies
    1. I thought the same until I saw that A2 stated that tanks like that of A5 are energy storage devices, even though they're not hydroelectric.

      Delete
    2. nice point... so I didn't get it. at least, as I used the same argument for claim 5, I hope it will work...

      Delete
  48. I made several comments above about A5 being CPA for claim 5 and it seems I was wrong.
    In fact, it's bad both ways. On one hand A5 has anti-buckling but the statement about anti-buckling in A5 (par 010 ?) is pretty confuse because. This statement is preceded by another statement saying that the tank does not withstand hydrostatic pressure. So the tank in A5 is not suitable for being emptied underwater because it will be destroyed by hydrostatic. But the definition of "anti-buckling" in A1 is different. Nevertheless, this drawback apparently renders the tank of A5 unsuitable for an electric energy buffer underwater . On the other hand , the combination C4+C5 differs from A4 by the wire ropes going all the way long through the walls and holding the pipe segments together. The "official" solution seems to be that A4 has already longitudinal cavities and the skilled man should "simply" remove the screws and insert wire ropes 14 of A5 inside these cavities. A4 discloses the longitudinal cavities in par 003 BUT DOES NOT SHOW ANY OF THEM ON FIGURE 1. The consequence is that said cavities may be inside the walls of the pipe segments with no connection to outside and among each other, which makes them unsuitable for the insertion of the wire ropes (because the wire ropes go from one pipe end segment to to the other pipe end segment). A good candidate must GUESS what the Committee actually meant. I understand that the information given in the paper must not necessarily be true, however if engineering basics are misleading (say holes are in walls but you don't see the holes in the cross-section drawing = misleading) then one has a better chance with 6/49 than getting 50 in C.

    ReplyDelete
  49. I forgot also about the elastomeric sealings of A4 first embodiment. One incorporating the A5 ropes into A4 must as well drill through the sealings... not only through the adjacent parts of the concrete walls.. I mean, so much fiction ..

    ReplyDelete
  50. I did an Art 54)(3) with A4 on C1 and then use A4 +A6 for C2. That does work

    ReplyDelete
  51. For those who attacked the novelty of claim4 with A5. How did you make correspond the tank of A5 with the reservoirs of claim 4 ? A1 defines a reservoir as any compartment surrounded by a wall out of which water can be PUMPED out and let in. In A5 water cannot be pumped out, only the petroleum can. In A5 the water is PUSHed out. How did you make these elements correspond? Was there a document saying reservoirs and tanks are the same thing?

    ReplyDelete
    Replies
    1. A5:[8] teaches that a "fluid (e.g. petroleum)" may be pumped in or out. I also found that confusing, because A1 indeed only defines the reservoir with respect of pumping water. I searched for a definition of "water = fluid", but didnt find one. However, at this point I found it so trivial to equate fluid to water that I dared applying this "technical knowledge"

      (In C2014 you had to assume that "clover-shaped" was the "shape of a flower" without any definition in the paper, sometimes it seems ok to use general knowledge...in C2018 however, the paper gave a definition that cows are dairy animals, which had to be cited...the line between general knowdledge and disallowed technical reasoning seems to be a sharp edge)

      Delete
    2. I agree that a A54(2)attack using A5 was probably intended. However I think that the paper was poorly drafted and as a result, lacking clear definitions which made the whole thing more difficult than previous years.The paper was also highly technical and heavily mechanical. For a chemist it created time management issues.

      Delete
  52. Hi Sander, should the validity of A4 as Art.54(3) against claim 1 not be checked, for the first application issue (Art.87(4))? Because the assignee of A3 is the same as the patentee of A4. I think it was worth to prove the invention of A4 is not the same as that of A3, otherwise the priority would not be valid for A4 which is an EP application, which would render A4 non prejudicial even in the sense of Art.54(3) against Claim 1. Best regards

    ReplyDelete
    Replies
    1. The priority of A4 was probably invalid, because it was filed on 5 Oct 14 and claimed priority of 2 Oct 13, and 2 Oct 14 was a Thursday. It was not filed within 1 year. I wondered though, if this was intentional or if they just mixed up the numbers.

      Anyways, A4 was filed on 05 Oct 14, thus before the effective date 11 Mar 15 of claim 1 and published thereafter on 6 Apr 15.

      A4 is prior art under Art. 54(3) EPC for claim 1, regardless whether the priority of A4 is valid or not.

      However, since A4 is not even novelty-destroying for claim 1, I found this all very unnecessary and misleading.

      Delete
    2. No calendars with paper C...

      Delete
  53. I agree that it seems the examiners were looking for a novelty attack for claim 4 based on A5 due to the definition given to "Buckling resistance" in A1 [4]:

    "Buckling resistance according to this invention means that at the deployment depth the reservoir does not collapse irrespective of the amount of water pumped out of the compartment."

    and the specification in A5 that:

    "The tank will not collapse, irrespective of the amount of water pushed away by the fluid."

    However, claim 4 requires "a structure providing buckling resistance", which is clearly intended to impart a technical limitation on the structure itself - i.e. that the structure prevents buckling when subject to differential pressure. This does not seem to be the case with A5, because A5 states that "the tank's walls are not built to resist forces arising from hydrostatic pressure".

    There is therefore no structure in A5 which is buckling resistant. It does buckle because of the way it functions never results in a differential pressure - it has nothing to do with the technical features of the structure. As in the first sentence of A5: [10], "the inside pressure is always identical to the outside pressure".

    If "buckling resistant" does indeed encompass this structure anything is "buckling resistant" because even a balloon would not buckle if it was used in such a way that no differential pressure was imparted on the structure.

    Overall, as with many other papers we have seen this year, the paper seems overly complex and has not been though through in detail.

    ReplyDelete
  54. Thank you very much for your effort for sharing the analysis.
    Regarding claim 6(5); [23] does not refer to a preferred range; rather to an ALTERNATIVE range. The cited GL H-IV 2.4 does not apply (it also has no case law cited in it)...
    The attack based on A123(2) on this claim 6(5) is a valid one. It was mentioned earlier that such an objection is weak because we can overcome it by an amendment. This logic does not work because every objection can be overcome. In particular novelty objections can easily be overcome by amendment e.g. from the description, drawings or claims...

    ReplyDelete
    Replies
    1. Thanks, I've changed it. The section is in the second paragraph of H-IV 2.4 but all case law has been removed.
      It used to be: H-IV 2.4 In the case of a disclosure of both a general and a preferred range a combination of the preferred disclosed narrower range and one of the part-ranges lying within the disclosed overall range on either side of the narrower range may be derivable from the original disclosure of the application.(see T925/98 citing T 2/81, OJ EPA 1982, 394, point 3; T 201/83, OJ EPA 1984, 481; and also T 53/82, T 571/89, T 656/92, T 522/96 and T 947/96, not published, but which all refer to T 2/81).

      Delete
    2. Thank you Jelle.

      But doesn't T2/81 require that he narrower range is a preferred one, i.e., that the narrow range is a further preferred embodiment of the general range? This should not be the case here since [23] discloses that the range of [22] could alternatively be 23-35. So 23-35 is not a preferred range of the range in [22].

      Delete
  55. Quotation of the EPO in december 2020:
    http://documents.epo.org/projects/babylon/eponot.nsf/0/66EB601464EC7BECC12586320050164F/$FILE/ExamPapers%20EQE%202021_EN.pdf

    So candidates were given the same number of documents to study for only 3 hours in 2021 in C1. This leaves about 2 hours or less to analyze the client letter plus 6 technically difficult documents for non specialized engineers in the subject matter. It was 5.5 hours in total in 2019, so about 3.5 hours for analyze.
    The syllabus and character has clearly changed. The preparation for the 2019 paper could not help for 2021. Candidates had to invent at the time of the exam another day of working.
    The cancelation of the 2021 paper has legal basis.

    ReplyDelete
  56. correction:
    Candidates had to invent at the time of the exam another way of working.

    ReplyDelete

Post a Comment

1 – 200 of 309 comments Newer Newest